[obm-l] RES:

2004-06-12 Thread David M. Cardoso

7(k-1),7k,7(k+1)

7(3k) = 273
k = 13

Maior ( 84,91,98 ) = 98 (par não-multiplo de 4)
Serah q ta certo?


> -Mensagem original-
> De: [EMAIL PROTECTED] 
> [mailto:[EMAIL PROTECTED] Em nome de elton 
> francisco ferreira
> Enviada em: sábado, 12 de junho de 2004 20:22
> Para: [EMAIL PROTECTED]
> Assunto: 
> 
> A soma de três múltiplos consecutivos de 7 é 273. O maior 
> desses números é um número:
> 
> por
> impar
> multiplo de 3
> multiplo de 4
> 
> 
> __
> 
> Participe da pesquisa global sobre o Yahoo! Mail: 
> http://br.surveys.yahoo.com/global_mail_survey_br
> ==
> ===
> Instruções para entrar na lista, sair da lista e usar a lista 
> em http://www.mat.puc-rio.br/~nicolau/olimp/obm-l.html
> ==
> ===
> 


=
Instruções para entrar na lista, sair da lista e usar a lista em
http://www.mat.puc-rio.br/~nicolau/olimp/obm-l.html
=


RES: [obm-l] Soma dos Quadrados...

2004-06-13 Thread David M. Cardoso

> imaginei x^2 - y^2 = 21
> tentei desmembrar ( x + y ) ( x - y ) = 21 , mas nao consegui 
> relacionar com x^2 + y^2 ...

(x+y)(x-y) = 3.7 = 7.3 = 1.21 = 21.1

1) (x+y) = 3
   (x-y) = 7

2) (x+y) = 7
   (x-y) = 3

3) (x+y) = 1
   (x-y) = 21

4) (x+y) = 21
   (x-y) = 1

De (1)
vc tira 2x = 10 -> x=5, y=(-2) --> x^2 - y^2 = 29

=
Instruções para entrar na lista, sair da lista e usar a lista em
http://www.mat.puc-rio.br/~nicolau/olimp/obm-l.html
=


RES: [obm-l] Problema envolvendo sistema linear

2004-06-15 Thread David M. Cardoso

> 
> L = nº laranjas
> P = nº peras
> X = nº pessoas
> 
> Faça:
> 3L = P
> 5X = L
> 8X + 21 = P
> 

Serei a calculadora:

-3L = -P
8x + 21 = P

8x - 3L = -21
5X - L  = 0 (vezes -3)
-15x +3L = 0

-7X = -21 ==> X = 3
  ==> L = 15
  ==> P = 45


=
Instruções para entrar na lista, sair da lista e usar a lista em
http://www.mat.puc-rio.br/~nicolau/olimp/obm-l.html
=


[obm-l] Problemas com binomiais

2004-06-24 Thread David M. Cardoso

Oi pessoal,

Gostaria da ajuda de voces com 2 questoes que não consigo fazer:
(qualquer comentario/ideia vai ajudar, eu não consigo sair do canto nessas
questoes)

http://www.suati.com.br/david/questao3.29.gif
http://www.suati.com.br/david/questao3.32.gif

[]'s
David


=
Instruções para entrar na lista, sair da lista e usar a lista em
http://www.mat.puc-rio.br/~nicolau/olimp/obm-l.html
=


RE: [obm-l] Problemas com binomiais

2004-06-24 Thread David M. Cardoso

ok... entendido.. obrigado! :) 

> -Mensagem original-
> De: [EMAIL PROTECTED] 
> [mailto:[EMAIL PROTECTED] Em nome de Leandro Lacorte Recova
> Enviada em: quinta-feira, 24 de junho de 2004 12:42
> Para: [EMAIL PROTECTED]
> Assunto: RE: [obm-l] Problemas com binomiais
> 
> Na segunda desigualdade, tente expandir o binomio (1+2)^n. 
> Voce encontrara a resposta imediato ! 

=
Instruções para entrar na lista, sair da lista e usar a lista em
http://www.mat.puc-rio.br/~nicolau/olimp/obm-l.html
=


RE: [obm-l] Problemas com binomiais

2004-06-24 Thread David M. Cardoso

Cara.. muuuito obrigado..
perfeito mesmo!!! entendi. :D 

> -Mensagem original-
> De: [EMAIL PROTECTED] 
> [mailto:[EMAIL PROTECTED] Em nome de claudio.buffara
> Enviada em: quinta-feira, 24 de junho de 2004 13:07
> Para: obm-l
> Assunto: Re:[obm-l] Problemas com binomiais
> 
> > Oi pessoal,
> > 
> > Gostaria da ajuda de voces com 2 questoes que não consigo fazer:
> > (qualquer comentario/ideia vai ajudar, eu não consigo sair do canto 
> > nessas
> > questoes)
> > 
> > http://www.suati.com.br/david/questao3.29.gif
>  
> O primeiro eh provar que n^k/k^k <= Binom(n,k) <= n^k/k!.
>  
> A segunda desigualdade eh mais facil:
> Binom(n,k) = n*(n-1)*...*(n-k+1)/k! <= n*n*...*n/k! = n^k/k!.
>  
> Com relacao a primeira, repare que se n = k, entao:
> n^k/k^k = Binom(n,k) = 1;
>  
> se n > k = 1, entao:
> n^k/k^k = Binom(n,k) = n;
>  
> finalmente, se n > k > 1, entao:
> n/k < (n-1)/(k-1) < (n-2)/(k-2) <  < (n-k+2)/2 < (n-k+1)/1
>  
> Basta provar a primeira desigualdade desta sequencia:
> n > k ==>
> n-1 > k-1 ==>
> 1/(n-1) < 1/(k-1) ==>
> 1 + 1/(n-1) < 1 + 1/(k-1) ==>
> n/(n-1) < k/(k-1) ==>
> n/k < (n-1)/(k-1)
>  
>  
> > http://www.suati.com.br/david/questao3.32.gif
>  
> Binom(n,0) + 2*Binom(n,1) + 2^2*Binom(n,2) + ... + 2^n*Binom(n,n) =
> (1 + 2)^n = 3^n.
>  
> Uma demonstracao combinatoria seria a seguinte:
> Temos um cartao de loteria esportiva com n jogos, cada um dos 
> quais com 3 alternativas (vitoria de um time, vitoria do 
> outro ou empate).
> De quantas maneiras podemos preenche-lo?
>  
> Obviamente, a resposta eh 3^n.
>  
> Por outro lado, poderiamos raciocinar da seguinte forma:
> Para cada k (0 <= k <= n), podemos preencher o cartao com k 
> empates e os demais n-k jogos com vitoria de um dois dois 
> times. Assim, para cada k, teremos:
> 1) Escolha dos jogos em que marcaremos empate:
> isso pode ser feito de Binom(n,k) formas diferentes.
>  
> 2) Em cada um dos (n-k) jogos em que marcaremos vitoria de 
> algum time, poderemos escolher o time de 2 maneiras. Logo, 
> poderemos marcar vitoria de 2^(n-k) formas diferentes.
>  
> Somando de k = 0 ateh k = n, acharemos o numero total de 
> maneiras de preencher o cartao:
> Binom(n,0)*2^n + Binom(n,1)*2^(n-1) + ... + Binom(n,n-1)*2^1 
> + Binom(n,n)*1, ou, levando em conta que Binom(n,k) = Binom(n,n-k):
> Binom(n,0)*1 + Binom(n,1)*2 + ... + Binom(n,n-1)*2^(n-1) + 
> Binom(n,n)*2^n.
>  
> Mas sabemos que esse numero eh igual a 3^n.
> Logo, a identidade estah provada.
>  
> []s,
> Claudio.
>  
>  
> 


=
Instruções para entrar na lista, sair da lista e usar a lista em
http://www.mat.puc-rio.br/~nicolau/olimp/obm-l.html
=


RES: [obm-l] UM PROBLEMA PECULIAR!

2004-07-14 Thread David M. Cardoso
 

> -Mensagem original-
> De: [EMAIL PROTECTED] 
> [mailto:[EMAIL PROTECTED] Em nome de [EMAIL PROTECTED]
> Enviada em: quarta-feira, 14 de julho de 2004 19:20
> Para: [EMAIL PROTECTED]
> Assunto: [obm-l] UM PROBLEMA PECULIAR!
> 
> OK! Artur, Rogério e demais colegas! vejam outro problema que 
> tenho dúvidas!!!
> 
> Cada um dos três envelopes A, B e C contém duas notas. Ao 
> todo, há três notas de
> $5 e três notas de $10, totalizando os $45 da soma das 
> importancias indicadas nos envelopes. Mas cuidado! Todos os 
> envelopes estão marcados incorretamente.
> Sem ver o conteúdo dos envelopes e retirando uma única nota 
> para ler seu valor, descubra qual o conteúdo de cada envelope.
> 
> ABC
>$20  $10  $15
(i)  5+5  5+10  10+10
  ou   ou ou
(ii) 10+5 10+10  5+5

Temos apenas duas possibilidades.

Se tirarmos uma nota de C, temos:
Se a nota for $10, acontece (i)
Se a nota for $5, acontece (ii)


=
Instruções para entrar na lista, sair da lista e usar a lista em
http://www.mat.puc-rio.br/~nicolau/olimp/obm-l.html
=


[obm-l] RES: [obm-l] [obm-l] Máximos e Mínimos

2004-07-14 Thread David M. Cardoso

> 
> 2) Se 3x + 4y = 100, qual é o valor mínimo de Sqrt (x^2 + ^y^2).

y=(100-3x)/4

f(x) = sqrt(x^2 + y^2) = sqrt(16x^2 + (100-3x)^2) / 4
f(x) = sqrt(16x^2 + 1 - 600x + 9x^2) / 4 =
 = sqrt(25x^2 - 600x + 1)/4 = (5/4)*sqrt(x^2 - 24x + 400)

Basta fazer a derivada e igualar a zero.
Sou novo em calculo mas acho q o (5/4) pode descartar,
e deve ter jeito melhor de fazer a derivada, mas bem.. eu fiz assim:

g(x) = f(x) * f(x) = x^2 - 24x + 400
g'(x) = 2x - 24 = 2*f'(x)*f(x)

f'(x) = 0 = (2x - 24)/2*f(x)

x = 12 eh solução

f(12) = ... contas ... = 20

20 eh o valor minimo.


=
Instruções para entrar na lista, sair da lista e usar a lista em
http://www.mat.puc-rio.br/~nicolau/olimp/obm-l.html
=


[obm-l] RES: [obm-l] Função Exponencial

2004-07-20 Thread David M. Cardoso

f(x) = e^x
g(x) = x

Pode ser assim?
 

> -Mensagem original-
> De: [EMAIL PROTECTED] 
> [mailto:[EMAIL PROTECTED] Em nome de Lista OBM
> Enviada em: terça-feira, 20 de julho de 2004 14:37
> Para: [EMAIL PROTECTED]
> Assunto: [obm-l] Função Exponencial
> 
> Gostaria de saber se existe duas funções reais f e g tais que 
> (fog)(x) = e^x.
>  
> Grato, Éder. 
> 
> 
> 
> Yahoo! Mail 
>   agora ainda melhor: 100MB, anti-spam e antivírus > grátis!
> 


=
Instruções para entrar na lista, sair da lista e usar a lista em
http://www.mat.puc-rio.br/~nicolau/olimp/obm-l.html
=


[obm-l] Problema Subconjuntos

2004-07-20 Thread David M. Cardoso


Olá,

Alguem pode me ajudar? Não consegui resolver o seguinte problema:

"Quantos subconjuntos o conjunto {1,2,3,...,n} tais que não contêm três
inteiros consecutivos?"

A dica dada na questão é: "Encontre uma recorrência." Porém, qualquer
solução (sem/com recorrência) vai ajudar.

[]'s
David


=
Instruções para entrar na lista, sair da lista e usar a lista em
http://www.mat.puc-rio.br/~nicolau/olimp/obm-l.html
=


[obm-l] Problema de Divisibilidade / Primos

2004-07-20 Thread David M. Cardoso

Mais duas questoes que não consigo me mecher:

Quantos inteiros existem que não são divisíveis por qualquer que seja o
primo maior que 20 e não são divisiveis por qualquer que seja o primo?

[]'s
David


=
Instruções para entrar na lista, sair da lista e usar a lista em
http://www.mat.puc-rio.br/~nicolau/olimp/obm-l.html
=


RES: [obm-l] Problema de Divisibilidade / Primos

2004-07-20 Thread David M. Cardoso

Droga droga droga !!!
Na pressa, errei o enunciado da questão!
Mil desculpas!

Segue o enunciado correto:

"Quantos inteiros existem que não são divisíveis por qualquer que seja o
primo maior que 20 e não são divisíveis pelo quadrado de qualquer que seja o
primo?"

Puxa vida... tenho prova amanha cedo, vou tentar tirar minhas duvidas de
ultima hora, tenho a sorte de voces existirem e ainda erro o enunciado da
questao... :~(

[]'s
David

> -Mensagem original-
> De: [EMAIL PROTECTED] 
> [mailto:[EMAIL PROTECTED] Em nome de Bruno França dos Reis
> Enviada em: terça-feira, 20 de julho de 2004 18:53
> Para: [EMAIL PROTECTED]
> Assunto: Re: [obm-l] Problema de Divisibilidade / Primos
> 
> -BEGIN PGP SIGNED MESSAGE-
> Hash: SHA1
> 
> On Tuesday 20 July 2004 18:26, David M. Cardoso wrote:
> > Mais duas questoes que não consigo me mecher:
> >
> > Quantos inteiros existem que não são divisíveis por 
> qualquer que seja 
> > o primo maior que 20 e não são divisiveis por qualquer que 
> seja o primo?
> 
> a) infinitos: 2^n não é divisível por qualquer que seja o 
> primo maior que 20, pois é divisível apenas pelo primo 2, 
> qualquer que seja n natural.
> 
> b) apenas o 1, pois qualquer outro número é divisível por ao 
> menos um primo: 
> se ele for composto, sabemos que ele é múltiplo de primos, e 
> se ele é primo, ele é divisível por si próprio, um número 
> primo. Já o 1 é divisível apenas por 1, que não é primo (e 
> não me venham com essa de que 1 é primo também!)
> 
> acho que é isso!
> 
> abraço
> 
> - --
> Bruno França dos Reis
> brunoreis at terra com br
> icq: 12626000
> gpg-key: 
> http://planeta.terra.com.br/informatica/brunoreis/brunoreis.key
> 
> -BEGIN PGP SIGNATURE-
> Version: GnuPG v1.2.4 (GNU/Linux)
> 
> iD8DBQFA/ZREsHdDIT+qyroRAhQFAKDOZm/uCMp38TYe+uXT2rL+lkNPWQCfWTdb
> iMrCfq37UfF/7EZvrP6Qm3g=
> =qpSy
> -END PGP SIGNATURE-
> 
> ==
> ===
> Instruções para entrar na lista, sair da lista e usar a lista 
> em http://www.mat.puc-rio.br/~nicolau/olimp/obm-l.html
> ==
> ===
> 


=
Instruções para entrar na lista, sair da lista e usar a lista em
http://www.mat.puc-rio.br/~nicolau/olimp/obm-l.html
=


RES: [obm-l] Problema de Divisibilidade / Primos

2004-07-20 Thread David M. Cardoso
Aeee ... acabei de pensar na solucao, não sei se ta certo:

se n é o produto de k primos (i<=k<=8), entao
n = p_1 * p_2 * p_3 * ... * p_k

tal que p_i < 20 (1 <= i <= k)
entao p_i pertence ao conjunto dos primos menores que 20 {
2,3,5,7,11,13,17,19 }
queremos contar os subconjuntos desse conjunto... menos o vazio..

temos entao 2^8 - 1 numeros deste tipo.

Ta certo?

[]'s
David

> -Mensagem original-
> De: [EMAIL PROTECTED] 
> [mailto:[EMAIL PROTECTED] Em nome de David M. Cardoso
> Enviada em: terça-feira, 20 de julho de 2004 20:11
> Para: [EMAIL PROTECTED]
> Assunto: RES: [obm-l] Problema de Divisibilidade / Primos
> 
> 
> Droga droga droga !!!
> Na pressa, errei o enunciado da questão!
> Mil desculpas!
> 
> Segue o enunciado correto:
> 
> "Quantos inteiros existem que não são divisíveis por qualquer 
> que seja o primo maior que 20 e não são divisíveis pelo 
> quadrado de qualquer que seja o primo?"
> 
> Puxa vida... tenho prova amanha cedo, vou tentar tirar minhas 
> duvidas de ultima hora, tenho a sorte de voces existirem e 
> ainda erro o enunciado da questao... :~(
> 
> []'s
> David
> 
> > -Mensagem original-
> > De: [EMAIL PROTECTED]
> > [mailto:[EMAIL PROTECTED] Em nome de Bruno França 
> dos Reis 
> > Enviada em: terça-feira, 20 de julho de 2004 18:53
> > Para: [EMAIL PROTECTED]
> > Assunto: Re: [obm-l] Problema de Divisibilidade / Primos
> > 
> > -BEGIN PGP SIGNED MESSAGE-
> > Hash: SHA1
> > 
> > On Tuesday 20 July 2004 18:26, David M. Cardoso wrote:
> > > Mais duas questoes que não consigo me mecher:
> > >
> > > Quantos inteiros existem que não são divisíveis por
> > qualquer que seja
> > > o primo maior que 20 e não são divisiveis por qualquer que
> > seja o primo?
> > 
> > a) infinitos: 2^n não é divisível por qualquer que seja o 
> primo maior 
> > que 20, pois é divisível apenas pelo primo 2, qualquer que seja n 
> > natural.
> > 
> > b) apenas o 1, pois qualquer outro número é divisível por 
> ao menos um 
> > primo:
> > se ele for composto, sabemos que ele é múltiplo de primos, 
> e se ele é 
> > primo, ele é divisível por si próprio, um número primo. Já o 1 é 
> > divisível apenas por 1, que não é primo (e não me venham 
> com essa de 
> > que 1 é primo também!)
> > 
> > acho que é isso!
> > 
> > abraço
> > 
> > - --
> > Bruno França dos Reis
> > brunoreis at terra com br
> > icq: 12626000
> > gpg-key: 
> > http://planeta.terra.com.br/informatica/brunoreis/brunoreis.key
> > 
> > -BEGIN PGP SIGNATURE-
> > Version: GnuPG v1.2.4 (GNU/Linux)
> > 
> > iD8DBQFA/ZREsHdDIT+qyroRAhQFAKDOZm/uCMp38TYe+uXT2rL+lkNPWQCfWTdb
> > iMrCfq37UfF/7EZvrP6Qm3g=
> > =qpSy
> > -END PGP SIGNATURE-
> > 
> > ==
> > ===
> > Instruções para entrar na lista, sair da lista e usar a lista em 
> > http://www.mat.puc-rio.br/~nicolau/olimp/obm-l.html
> > ==
> > ===
> > 
> 
> 
> ==
> ===
> Instruções para entrar na lista, sair da lista e usar a lista 
> em http://www.mat.puc-rio.br/~nicolau/olimp/obm-l.html
> ==
> ===
> 


=
Instruções para entrar na lista, sair da lista e usar a lista em
http://www.mat.puc-rio.br/~nicolau/olimp/obm-l.html
=


RES: [obm-l] Problema Subconjuntos

2004-07-20 Thread David M. Cardoso

Cara, muito obrigado..
Sendo que ta dando trabalho pra eu entender algumas coisas,
como "teremos T[n-3] - T[n-4] subconjuntos com os elementos n-1 e n-2"..
hora eu penso que entendi, hora eu não entendo mais e fico tentando lembrar
pq eu fico entendido antes, talvez seja o nervosismo, talvez seja apenas
porque o raciocinio eh complicado demais pra mim..

Outra duvida que tenho é se é possível transformar a recorrência num
"polinomiozinho" em função de n ou se uma resposta desse tipo já esta
completa o suficiente..

[]'s
David

> -Mensagem original-
> De: [EMAIL PROTECTED] 
> [mailto:[EMAIL PROTECTED] Em nome de Helder Suzuki
> Enviada em: terça-feira, 20 de julho de 2004 19:30
> Para: [EMAIL PROTECTED]
> Assunto: Re: [obm-l] Problema Subconjuntos
> 
> vamos ver, seguindo a dica de usar recorrencia
> 
> se T[n] for igual ao numero de subconjuntos do conjunto {1, 
> 2, ..., n} que nao contem 3 inteiros consecutivos.
> temos que:
> T[0] = 1
> {}
> 
> T[1] = 2
> {} e {1}
> 
> T[2] = 4
> {}, {1},
> {2} e {1, 2}
> 
> T[3] = 7
> {}, {1}, {2}, {1, 2},
> {3}, {1, 3}, {2, 3}
> 
> T[4] = 13
> {}, {1}, {2}, {3}, {1, 2}, {1, 3}, {2, 3}, {4}, {1, 4}, {2, 
> 4}, {3, 4}, {1, 2, 4}, {1, 3, 4}
> 
> bom, suponha que sabemos o valor de T[n-1], T[n-2], ..., 
> T[1]; como podemos achar T[n] em funcao de T[n-1]? humm...
> considere todos subconjuntos de {1, 2, 3, 4, ..., n-1} que 
> satisfazem a condicao do enunciado.
> se adicionarmos um elemento n, em quais desses subconjuntos o 
> n pode entrar e quais ele nao pode(para manter a condicao do 
> enunciado)?
> se n nao pode entrar em X subconjuntos, temos que T[n] = 
> T[n-1] + T[n-1] - X T[n] = 2*T[n-1] - X mas X eh o numero de 
> subconjuntos que tem os elementos
> n-1 e n-2.
> 
> imagine que temos os subconjnutos de {1, 2, ..., n-3} e 
> queremos adicionar os elementos n-1 e n-2 a esses 
> subconjuntos ao mesmo tempo, nesse caso só nao poderemos 
> adicionar n-1 e n-2 aos subconjuntos que tem o elemento n-3, 
> entao teremos T[n-3] - T[n-4] subconjuntos com os elementos n-1 e n-2:
> X = T[n-3] - T[n-4]
> 
> entao nossa recorrencia fica:
> T[n] = 2*T[n-1] - T[n-3] + T[n-4]
> 
> []'s,
> Helder
> 
> --- "David M. Cardoso" <[EMAIL PROTECTED]>
> escreveu: > 
> > 
> > Olá,
> > 
> > Alguem pode me ajudar? Não consegui resolver o seguinte problema:
> > 
> > "Quantos subconjuntos o conjunto {1,2,3,...,n} tais que não contêm 
> > três inteiros consecutivos?"
> > 
> > A dica dada na questão é: "Encontre uma recorrência." 
> Porém, qualquer 
> > solução (sem/com recorrência) vai ajudar.
> > 
> > []'s
> > David
> 
> 
>   
>   
>   
> ___
> Yahoo! Mail agora com 100MB, anti-spam e antivírus grátis!
> http://br.info.mail.yahoo.com/
> ==
> ===
> Instruções para entrar na lista, sair da lista e usar a lista 
> em http://www.mat.puc-rio.br/~nicolau/olimp/obm-l.html
> ==
> ===
> 


=
Instruções para entrar na lista, sair da lista e usar a lista em
http://www.mat.puc-rio.br/~nicolau/olimp/obm-l.html
=


RES: RES: [obm-l] Problema de Divisibilidade / Primos

2004-07-20 Thread David M. Cardoso

Realmente.. realmente.. o vazio conta como o numero 1..
ok .. obrigado!

[]'s
David

> -Mensagem original-
> De: [EMAIL PROTECTED] 
> [mailto:[EMAIL PROTECTED] Em nome de Bernardo 
> Freitas Paulo da Costa
> Enviada em: terça-feira, 20 de julho de 2004 21:29
> Para: [EMAIL PROTECTED]
> Assunto: Re: RES: [obm-l] Problema de Divisibilidade / Primos
> 
> Oi, David,
> 
> Enumere os primos menores do que 20:
> 2, 3, 5, 7, 11, 13, 17, 19: são 8.
> 
> Um número que satisfaça as condições do enunciado pode ter, 
> no máximo, um de cada um destes fatores, pela segunda parte, 
> e nenhum outro fator, pela primeira parte.
> Assim, temos um problema de combinatória, agora:
> quantos números podemos formar utilizando apenas o produto de 
> 8 primos, onde não podemos incluir um primo duas vezes. Ou, 
> mais combinatória ainda, quantos subconjuntos de um conjunto 
> de 8 elementos existem?
> Para ver que as soluções são iguais, associe a cada 
> subconjunto o número correspondente ao produto de seus 
> elementos, e ao subconjunto vazio o número 1 (eis aqui mais 
> uma boa justificativa para termos um produtório vazio valendo 1!!)
> 
> Bom, para este problema a resposta é conhecida: vale 2^8 = 256.
> Pronto, são 256 números.
> 
> Abraços,
> Bernardo Costa
> 
> 
> On Tue, 20 Jul 2004, David M. Cardoso wrote:
> 
> > 
> > Droga droga droga !!!
> > Na pressa, errei o enunciado da questão!
> > Mil desculpas!
> > 
> > Segue o enunciado correto:
> > 
> > "Quantos inteiros existem que não são divisíveis por 
> qualquer que seja 
> > o primo maior que 20 e não são divisíveis pelo quadrado de qualquer 
> > que seja o primo?"
> > 
> > Puxa vida... tenho prova amanha cedo, vou tentar tirar 
> minhas duvidas 
> > de ultima hora, tenho a sorte de voces existirem e ainda erro o 
> > enunciado da questao... :~(
> > 
> > []'s
> > David
> > 
> > > -Mensagem original-
> > > De: [EMAIL PROTECTED]
> > > [mailto:[EMAIL PROTECTED] Em nome de Bruno 
> França dos Reis 
> > > Enviada em: terça-feira, 20 de julho de 2004 18:53
> > > Para: [EMAIL PROTECTED]
> > > Assunto: Re: [obm-l] Problema de Divisibilidade / Primos
> > > 
> > > -BEGIN PGP SIGNED MESSAGE-
> > > Hash: SHA1
> > > 
> > > On Tuesday 20 July 2004 18:26, David M. Cardoso wrote:
> > > > Mais duas questoes que não consigo me mecher:
> > > >
> > > > Quantos inteiros existem que não são divisíveis por
> > > qualquer que seja
> > > > o primo maior que 20 e não são divisiveis por qualquer que
> > > seja o primo?
> > > 
> > > a) infinitos: 2^n não é divisível por qualquer que seja o primo 
> > > maior que 20, pois é divisível apenas pelo primo 2, qualquer que 
> > > seja n natural.
> > > 
> > > b) apenas o 1, pois qualquer outro número é divisível por 
> ao menos 
> > > um primo:
> > > se ele for composto, sabemos que ele é múltiplo de 
> primos, e se ele 
> > > é primo, ele é divisível por si próprio, um número primo. 
> Já o 1 é 
> > > divisível apenas por 1, que não é primo (e não me venham 
> com essa de 
> > > que 1 é primo também!)
> > > 
> > > acho que é isso!
> > > 
> > > abraço
> > > 
> > > - --
> > > Bruno França dos Reis
> > > brunoreis at terra com br
> > > icq: 12626000
> > > gpg-key: 
> > > http://planeta.terra.com.br/informatica/brunoreis/brunoreis.key
> > > 
> > > -BEGIN PGP SIGNATURE-
> > > Version: GnuPG v1.2.4 (GNU/Linux)
> > > 
> > > iD8DBQFA/ZREsHdDIT+qyroRAhQFAKDOZm/uCMp38TYe+uXT2rL+lkNPWQCfWTdb
> > > iMrCfq37UfF/7EZvrP6Qm3g=
> > > =qpSy
> > > -END PGP SIGNATURE-
> > > 
> > > ==
> > > ===
> > > Instruções para entrar na lista, sair da lista e usar a lista em 
> > > http://www.mat.puc-rio.br/~nicolau/olimp/obm-l.html
> > > ==
> > > ===
> > > 
> > 
> > 
> > 
> ==
> > === Instruções para entrar na lista, sair da lista e usar a 
> lista em 
> > http://www.mat.puc-rio.br/~nicolau/olimp/obm-l.html
> > 
> ==
> > ===
> > 
> 
> ==
> ===
> Instruções para entrar na lista, sair da lista e usar a lista 
> em http://www.mat.puc-rio.br/~nicolau/olimp/obm-l.html
> ==
> ===
> 


=
Instruções para entrar na lista, sair da lista e usar a lista em
http://www.mat.puc-rio.br/~nicolau/olimp/obm-l.html
=


[obm-l] Problema - Primos

2004-07-20 Thread David M. Cardoso

Mais um problema não resolvido:

"Mostre que um número com 30 dígitos não pode ter mais que 100 fatores
primos."

[]'s
David


=
Instruções para entrar na lista, sair da lista e usar a lista em
http://www.mat.puc-rio.br/~nicolau/olimp/obm-l.html
=


[obm-l] Problema - Recorrência / Fibonacci

2004-07-20 Thread David M. Cardoso

Olá novamente,

Seja F_n a recorrência definida por F_(n+1) = F_n + F_(n-1).
Com F_1 = 1, F_2 = 1, ... (sequencia de fibonacci)

"Qual é o maior: 2^100 ou F_100 ?"

deu pra perceber, testando, que 2^100 é maior.
Ateh porque 2^(n+1) / 2^n = 2
Enquanto que F_(n+1) / F_(n) ~ 1,618 quando n é grande.

Mas não sei formalizar/mostrar que 2^100 é de fato o maior.

[]'s
David


=
Instruções para entrar na lista, sair da lista e usar a lista em
http://www.mat.puc-rio.br/~nicolau/olimp/obm-l.html
=


RES: [obm-l] Problema - Primos

2004-07-20 Thread David M. Cardoso
> 
> "Mostre que um número com 30 dígitos não pode ter mais que 
> 100 fatores primos."
> 

Bem.. talvez eu tenha feito, acho que eh soh mostrar que
Piso[Log_10[2^100]+1] = 31
e que portanto 2^100, que é o menor produto de 100 fatores primos, tem 31
dígitos.

[]'s
David


=
Instruções para entrar na lista, sair da lista e usar a lista em
http://www.mat.puc-rio.br/~nicolau/olimp/obm-l.html
=


[obm-l] Problema - Matemática Discreta

2004-07-20 Thread David M. Cardoso

Eu não sei em que tópico este problema se enquadra, por isso coloquei no
assunto a disciplina que tem relação com ele. Não consegui fazer:

"Existem (m-1)n + 1 pessoas na sala. Mostre que ou existem m pessoas que não
se conhecem mutuamente, ou existe uma pessoa que conhece pelo menos n
outras."

[]'s
David


=
Instruções para entrar na lista, sair da lista e usar a lista em
http://www.mat.puc-rio.br/~nicolau/olimp/obm-l.html
=


[obm-l] RES: [obm-l] Problema - Recorrência / Fibonacci

2004-07-20 Thread David M. Cardoso

Entendi.. entendi.. obrigado.

[]'s

> -Mensagem original-
> De: [EMAIL PROTECTED] 
> [mailto:[EMAIL PROTECTED] Em nome de Domingos Jr.
> Enviada em: terça-feira, 20 de julho de 2004 23:44
> Para: [EMAIL PROTECTED]
> Assunto: Re: [obm-l] Problema - Recorrência / Fibonacci
> 
> David M. Cardoso wrote:
> 
> >Olá novamente,
> >
> >Seja F_n a recorrência definida por F_(n+1) = F_n + F_(n-1).
> >Com F_1 = 1, F_2 = 1, ... (sequencia de fibonacci)
> >
> >"Qual é o maior: 2^100 ou F_100 ?"
> >
> >deu pra perceber, testando, que 2^100 é maior.
> >Ateh porque 2^(n+1) / 2^n = 2
> >Enquanto que F_(n+1) / F_(n) ~ 1,618 quando n é grande.
> >
> >Mas não sei formalizar/mostrar que 2^100 é de fato o maior.
> >
> Você pode provar o resultado por indução para todo n, veja:
> para n = 1, 2, F_n = 1 < 2^n
> 
> F_{n+1} = F_n + F{n-1} < 2^n + 2^{n-1} = 3*2^{n-1} < 
> 4*2^{n-1} = 2^{n+1}
> 
> e o resultado segue por indução.
> ==
> ===
> Instruções para entrar na lista, sair da lista e usar a lista 
> em http://www.mat.puc-rio.br/~nicolau/olimp/obm-l.html
> ==
> ===
> 


=
Instruções para entrar na lista, sair da lista e usar a lista em
http://www.mat.puc-rio.br/~nicolau/olimp/obm-l.html
=


RES: [obm-l] Moedas em Cofrinhos

2004-07-27 Thread David M. Cardoso

> -Mensagem original-
> De: [EMAIL PROTECTED] 
> [mailto:[EMAIL PROTECTED] Em nome de claudio.buffara
> Enviada em: domingo, 25 de julho de 2004 11:09
> Para: obm-l
> Assunto: [obm-l] Moedas em Cofrinhos
> 
> Oi, pessoal:
>  
> Um problema bonitinho:
>  
> Temos n moedas de R$ 1,00 e n moedas de R$ 0,25. 
> Moedas de mesma denominacao sao supostas indistinguiveis.
> De quantas maneiras podemos escolher n moedas (dentre as 2n 
> que temos) e distribui-las por dentre m cofrinhos (cofrinhos 
> podem ficar vazios)?
> a) Supondo os cofrinhos numerados de 1 a m.
> b) Supondo os cofrinhos indistinguiveis.
>  
> []s,
> Claudio.

Pessoal, ninguem vai responder esse? queria saber como faz..

Eu cheguei a tentar alguma coisa, desde que ele colocou.. mas não consegui
resolver e fiquei com vergonha de postar o que eu tinha pensado..

Como ta demorando, eu vo postar o q eu cheguei a fazer:

--

Escolher n moedas dentre as 2n que temos:

Vamos escolher k moedas (0<=k<=n) de R$1,00
e (n-k) moedas de R$0,25:
Cada escolha de k determina uma possibilidade. 

Para cada k escolhido, vamos distribuir as moedas entre os cofrinhos:

Distribuiremos primeiramente as k moedas de 1,00 e depois as
(n-k) entre os m cofrinhos.

Para a letra a)

Sabemos que se temos:
c_1 + c_2 + ... + c_m = x

Binom[x-1,m-1] é o numero de solucoes tal q cada cofrinho recebe pelo menos
1.
Vamos tomar 1 moeda "emprestada" de cada cofrinho, para que ao receber pelo
menos 1, ele esteja recebendo de volta a moeda q emprestou, podendo resultar
num recebimento de
nenhuma moeda.

Queremos as solucoes de:
c_1 + c_2 + ... + c_m = k+m ==> Binom[k+m-1,m-1]
c_1 + c_2 + ... + c_m = (n-k)+m ==> Binom[n-k+m-1,m-1]

Teremos Soma{0<=k<=n} Binom[k+m-1,m-1]*Binom[n-k+m-1,m-1]
xii... sei transformar isso em formula não.. :/


=
Instruções para entrar na lista, sair da lista e usar a lista em
http://www.mat.puc-rio.br/~nicolau/olimp/obm-l.html
=


Re: [obm-l] [PELEJA] Desafio do Google

2004-09-17 Thread David M. Cardoso

Como faz pra achar isso?
Apelação/Força bruta computacional?
E se for um problema te testar exaustivamente vários números (problema
computacional), como fazer pra encontrar a n-ésima casa decimal da constante
de euler?

[]'s
David


=
Instruções para entrar na lista, sair da lista e usar a lista em
http://www.mat.puc-rio.br/~nicolau/olimp/obm-l.html
=


RES: [obm-l] amigos do PONCE

2004-09-22 Thread David M. Cardoso

> > quanto vale a soma de todos os divisores de 720?

Bem.. eu tive uma ideia, não sei se ta certo:

720 | 2
360 | 2
180 | 2
90  | 2
45  | 3
15  | 3
5   | 5

2 -> 2, 2^2, 2^3, 2^4
3 -> 3, 3^2
5 -> 5

(1 + 2 + 2^2 + 2^3 + 2^4)(1 + 3 + 3^2)(1 + 5) = (1+2+4+8+16)(1+3+9)(1+5) = 2
418

Nós contamos o 1 como um divisor, então temos q subtrair 1 no final: 2 418 -
1 = 2 417. 


=
Instruções para entrar na lista, sair da lista e usar a lista em
http://www.mat.puc-rio.br/~nicolau/olimp/obm-l.html
=


RES: [obm-l] amigos do PONCE

2004-09-22 Thread David M. Cardoso
 

> quanto vale a soma de todos os divisores de 720?

720 |

=
Instruções para entrar na lista, sair da lista e usar a lista em
http://www.mat.puc-rio.br/~nicolau/olimp/obm-l.html
=


[obm-l] n-ésima casa decimal da constante "e" (do logaritmo natural)

2004-10-15 Thread David M. Cardoso

Como fazer para obter a n-ésima casa decimal da constante "e" (constante de
euler?) dado que se conhece (ou não) as k-ésimas casas decimais anteriores
(k < n) ?


=
Instruções para entrar na lista, sair da lista e usar a lista em
http://www.mat.puc-rio.br/~nicolau/olimp/obm-l.html
=


RES: [obm-l] Livro emPDF

2004-09-30 Thread David M. Cardoso

Eu também gostaria.. por favor, quando for mandar pra ele(s), coloca meu
e-mail na lista de destinatários: [EMAIL PROTECTED]

Abraço,
David

> -Mensagem original-
> De: [EMAIL PROTECTED] 
> [mailto:[EMAIL PROTECTED] Em nome de Danilo notes
> Enviada em: quinta-feira, 30 de setembro de 2004 15:19
> Para: [EMAIL PROTECTED]
> Assunto: Re: [obm-l] Livro emPDF
> 
> Ola Paulo , tentei te enviar o livro mas recebi uma mensagem de erro.
> O problema é que o livro é um arquivo com mais de 20 mega e 
> para ser enviado teria que ter um tamanho menor. Vou tentar 
> quebrar o arquivo em varios pedaços , se conseguir eu te envio ok?
>  
>Abs.
>  
>   Rivaldo
> 
> amurpe <[EMAIL PROTECTED]> wrote:
> 
>   prezado , Rbdantas
>   
>   Se não for incômodo pra você, ficarei muito grato, 
>   
>   pois será uma ajuda muito grande.
>   
>   Um abraço.
>   Paulo barclay
>   
>   -- Início da mensagem original ---
>   
>   De: [EMAIL PROTECTED]
>   Para: [EMAIL PROTECTED]
>   Cc: 
>   Data: Fri, 24 Sep 2004 17:55:38 -0300
>   Assunto: Re: [obm-l] Livro emPDF
>   
>   > Citando paulobarclay :
>   > eu tenho livro e posso lhe enviar se alguem ainda não 
>   o fez.
>   > 
>   > 
>   > 
>   > > Prezados, preciso adquirir o livro de algebra 
>   linear 
>   > > do Hoffman -Kunze.
>   > > No entanto soube( será que é boato ?) que existe 
>   uma 
>   > > versão em pdf desse livro circulando na Internet.
>   > > 
>   > > Se for realmente verdade, e se alguem da lista tem 
>   o 
>   > > livro ou sabe como obtê-lo, ! e puder me dar uma dica 
>   > > ficarei imensamente grato.
>   > > 
>   > > Desde já obrigado.
>   > > 
>   > > Paulo Barclay
>   > > 
>   > > 
>   > > 
>   > > 
>   
>   __
>   > > Acabe com aquelas janelinhas que pulam na sua tela.
>   > > AntiPop-up UOL - É grátis!
>   > > http://antipopup.uol.com.br/
>   > > 
>   > > 
>   > > 
>   > > 
>   
>   =
>   > > Instruções para entrar na lista, sair da lista e 
>   usar a lista em
>   > > http://www.mat.puc-rio.br/~nicolau/olimp/obm-l.html
>   > > 
>   
>   =
>   > > 
>   > 
>   > 
>   > 
>   > 
>   > -
>   > URI - Campus de Santo Angelo-RS
>   > http://www.urisan.tche.b! r
>   > 
>   > 
>   
>   =
>   > Instruções para entrar na lista, sair da lista e usar 
>   a lista em
>   > http://www.mat.puc-rio.br/~nicolau/olimp/obm-l.html
>   > 
>   
>   =
>   > 
>   
>   
> __
> 
>   Acabe com aquelas janelinhas que pulam na sua tela.
>   AntiPop-up UOL - É grátis!
>   http://antipopup.uol.com.br/
>   
>   
>   
>   
> ==
> ===
>   Instruções para entrar na lista, sair da lista e usar a lista em
>   http://www.mat.puc-rio.br/~nicolau/olimp/obm-l.html
>   
> ==
> ===
>   
> 
> 
> 
> Yahoo! Acesso Grátis 
>   - Internet rápida e grátis. Instale o discador agora!
> 


=
Instruções para entrar na lista, sair da lista e usar a lista em
http://www.mat.puc-rio.br/~nicolau/olimp/obm-l.html
=


[obm-l] RE: [obm-l] Discussão no MSN

2004-10-22 Thread David M. Cardoso

Legal... ;)

o meu é
[EMAIL PROTECTED] 

[]'s

> -Original Message-
> From: [EMAIL PROTECTED] 
> [mailto:[EMAIL PROTECTED] On Behalf Of Daniel Regufe
> Sent: Friday, October 22, 2004 9:18 AM
> To: [EMAIL PROTECTED]
> Subject: [obm-l] Discussão no MSN 
> 
> Bom dia pra todos da lista ... Eu gostaria de saber quem da 
> lista tem MSN ?? 
> Nós podemos discutir matemática no MSN tb né !
> 
> o  meu eh [EMAIL PROTECTED]
> []´
> Daniel Regufe
> 
> _
> MSN Messenger: converse com os seus amigos online.  
> http://messenger.msn.com.br
> 
> ==
> ===
> Instruções para entrar na lista, sair da lista e usar a lista 
> em http://www.mat.puc-rio.br/~nicolau/olimp/obm-l.html
> ==
> ===
> 


=
Instruções para entrar na lista, sair da lista e usar a lista em
http://www.mat.puc-rio.br/~nicolau/olimp/obm-l.html
=


RE: [obm-l] Probabilidade: Moeda grossa

2004-10-23 Thread David M. Cardoso

r/2 ?

Se R for o raio e h for a grossura,
a área da aresta teria que ser 1/3 da área total da moeda ?

2piRh / (2piRh + 2piR^2) = 1/3
h/(h + R) = 1/3
(h + R) = 3h
R = 2h
h = R/2

> -Original Message-
> From: [EMAIL PROTECTED] 
> [mailto:[EMAIL PROTECTED] On Behalf Of Fabio Niski
> Sent: Saturday, October 23, 2004 12:19 PM
> To: [EMAIL PROTECTED]
> Subject: [obm-l] Probabilidade: Moeda grossa
> 
> Quão grossa deve ser uma moeda para que o resultado de seu 
> lançamento resulte na aresta com probabilidade 1/3?
> 
> Quem conhece deixa os outros pensarem! :))
> 
> ==
> ===
> Instruções para entrar na lista, sair da lista e usar a lista 
> em http://www.mat.puc-rio.br/~nicolau/olimp/obm-l.html
> ==
> ===
> 


=
Instruções para entrar na lista, sair da lista e usar a lista em
http://www.mat.puc-rio.br/~nicolau/olimp/obm-l.html
=


[obm-l] Probabilidade - Dígitos aleatórios

2004-10-27 Thread David M. Cardoso

Tentei, tentei, tentei um pouco mais e não consegui encontrar uma solução:

Suponha que os n dígitos 1,2,3,...,n sejam escritos em ordem aleatória. Qual
é a probabilidade de que ao menos um dígito ocupe seu lugar próprio?

Alguém ajuda?

Abraço,
David


=
Instruções para entrar na lista, sair da lista e usar a lista em
http://www.mat.puc-rio.br/~nicolau/olimp/obm-l.html
=


[obm-l] RE: [obm-l] Probabilidade - Dígitos aleatórios

2004-10-27 Thread David M. Cardoso

Puxa.. não consegui encontrar a solução..
Encontrei isso aqui:
http://www.mat.puc-rio.br/~nicolau/olimp/obm-l.200212/msg00040.html.
A única coisa que eu vi eh que tem a ver com e^(-1).

Conheço pouco(nada)... Mas estava tentando fazer assim (ta errado):

Dada uma sequência qualquer, se P_k for a probabilidade de o k-ésimo dígito
não ser k,
eu encontrei, meio que intuitivamente, que P_k = 1 - (1/n).

Depois, pensei que se eu quero que todos sejam diferentes, eu deveria ter
P(n) = (1 - (1/n))^n.
E aí, pra inverter a situação, bastaria fazer 1 - P(n) no final... Mas
percebi que estava errado, pois pra n=3 é fácil ver que a probabilidade
seria 2/3.

Pra completar a questão, o exercício pede que você estude o limite desta
probabilidade quando n é grande.. Enfim... de qualquer forma, obrigado.

Abraço,
David

> -Original Message-
> From: [EMAIL PROTECTED] 
> [mailto:[EMAIL PROTECTED] On Behalf Of Johann Peter 
> Gustav Lejeune Dirichlet
> Sent: Wednesday, October 27, 2004 6:31 PM
> To: [EMAIL PROTECTED]
> Subject: Re: [obm-l] Probabilidade - Dígitos aleatórios
> 
> Bem, diversas vzes na lista foui discutido o problema 
> inverso: a probabilidade de nenhum digito estar em sua 
> posicao. Dai, procure nos servidores e acabou!
> 
> "David M. Cardoso" <[EMAIL PROTECTED]> wrote: 
> 
> 
>   Tentei, tentei, tentei um pouco mais e não consegui 
> encontrar uma solução:
>   
>   Suponha que os n dígitos 1,2,3,...,n sejam escritos em 
> ordem aleatória. Qual
>   é a probabilidade de que ao menos um dígito ocupe seu 
> lugar próprio?
>   
>   Alguém ajuda?
>   
>   Abraço,
>   David
>   
>   
>   
> ==
> ===
>   Instruções para entrar na lista, sair da lista e usar a lista em
>   http://www.mat.puc-rio.br/~nicolau/olimp/obm-l.html
>   
> ==
> ===
>   
> 
> 
> 
> Yahoo! Acesso Grátis 
> <http://br.rd.yahoo.com/mail/taglines/*http://br.acesso.yahoo.
com/>  - Internet rápida e grátis. Instale o discador agora!
> 


=
Instruções para entrar na lista, sair da lista e usar a lista em
http://www.mat.puc-rio.br/~nicolau/olimp/obm-l.html
=


[obm-l] RE: [obm-l] Probabilidade - Dígitos aleatórios

2004-10-28 Thread David M. Cardoso

Bem.. eu vou pensar um pouco sobre isso que vc fez.. mas ta meio dificil pra
mim..

Obrigado a todos..

[]s
David

> 
> A solução era C(n) = 1 - 1/2! + 1/3! - 1/4! + ... + 
> (-1)^(n+1)/n!. No limite quando n -> oo, isso tende para 1 - e^(-1).
> 
> Eu cheguei a fazer esse desenvolvimento por conta própria na 
> época (depois de vários equívocos, como sempre), mas ficou 
> extremamente extenso, usando diversas induções... Considerei 
> basicamente seqüências P(i), que correspondiam ao número de 
> permutações da matriz identidade que mantinham a i-ésima 
> linha inalterada e que não repetiam permutações já 
> consideradas anteriormente; era assim:
> 
> P(1) = (n-1)!
> P(2) = (n-1)! - (n-2)!
> P(3) = (n-1)! - 2*(n-2)! + (n-3)!
> ...
> P(k) = Somatório(i=0,k-1) (k-1)!/(i!*(k-i)!)*(-1)^i*(n-1-i)!
> 
> Somam-se os P(k) e dividi-se tudo por n! para obter a expressão C(n).
> 


=
Instruções para entrar na lista, sair da lista e usar a lista em
http://www.mat.puc-rio.br/~nicolau/olimp/obm-l.html
=


[obm-l] RE: [obm-l] Probabilidade - Dígitos aleatórios

2004-10-28 Thread David M. Cardoso

>
> Qual a probabilidade de que exatamente n-1 digitos ocupem o 
> seu lugar proprio?
> 

zero? :o

=
Instruções para entrar na lista, sair da lista e usar a lista em
http://www.mat.puc-rio.br/~nicolau/olimp/obm-l.html
=


[obm-l] RE: [obm-l] RE: [obm-l] Probabilidade - Dígitos aleatórios

2004-10-28 Thread David M. Cardoso

> 
> >
> > Qual a probabilidade de que exatamente n-1 digitos ocupem o 
> seu lugar 
> > proprio?
> > 
> 
> zero? :o

Desculpa o enunciado pouco esclarecedor(pouco é pouco?), mas é que não pode
aparecer dígito repetido.. aí se (n-1) dígitos ocupam seu lugar próprio, o
dígito que falta pôr é justamente o dígito correspondente a posição e aí
ficariam n dígitos ocupando seu lugar próprio.. são permutações de números
de n dígitos.. deu pra entender?


=
Instruções para entrar na lista, sair da lista e usar a lista em
http://www.mat.puc-rio.br/~nicolau/olimp/obm-l.html
=


[obm-l] Afinal de contas, qual a grossura da moeda?

2004-10-31 Thread David M. Cardoso

Ainda estou curioso pra saber como calcula a grossura da moeda pra que o
lançamento resulte sobre a aresta com probabilidade 1/3. Acho que já deu
tempo de pensarem (na verdade, acho que nem deram muita atenção). Você não
quer me matar de curiosidade. Quer?!


=
Instruções para entrar na lista, sair da lista e usar a lista em
http://www.mat.puc-rio.br/~nicolau/olimp/obm-l.html
=


[obm-l] Problemas - Vestibular

2003-12-05 Thread David M. Cardoso

Oi pessoal, sou novo na lista e gostaria da ajuda de vocês com
alguns problemas de vestibular que não consegui resolver.
Tenho certeza que vcs podem me ajudar. :-)

1)
   1
 = A + B*cbrt(2) + C*cbrt(4)
 1 + 3cbrt(2) + cbrt(4)

Determinar 41(A + 2B + C)
cbrt(x) = raiz cubica de x

--//--

2) Sendo A e B naturais definidos pela igualdade

A + Bsqrt(2) = (1 + sqrt(2))^1999
Sqrt(x) = raiz quadrada de x

0-0) A - Bsqrt(2) = (1 - sqrt(2))^1999
1-1) A^2 - 2B^2 = 1
2-2) 2A = (1 + sqrt(2))^1999 + (1-sqrt(2))^1999
3-3) 2A é o maior inteiro menor que (1 + sqrt(2))^1999
4-4) B é par.

--//--

3) Analisar as afirmações:

0-0) Os primos que dividem 2.3.5.7.11.13.17 + 1 são >= 19
1-1) Os numeros consecutivos 11!+2, 11!+3, 11!+4, 11!+5, ..., 11!+11
 são compostos.
2-2) O dígito das dezenas de 11^10 + 1 é 1
3-3) 2^98 divide 100!
4-4) 2^1999 + 1 é multiplo de 3

Bem... vou parar por aqui, não quero abusar da boa vontade da turma.

David M. Cardoso
[EMAIL PROTECTED]


=
Instruções para entrar na lista, sair da lista e usar a lista em
http://www.mat.puc-rio.br/~nicolau/olimp/obm-l.html
=


[obm-l] Problema - Combinatória

2003-12-06 Thread David M. Cardoso

Gostaria da ajuda de vcs:
http://www.suati.com.br/david/questao15.gif

=
Instruções para entrar na lista, sair da lista e usar a lista em
http://www.mat.puc-rio.br/~nicolau/olimp/obm-l.html
=


[obm-l] RE: [obm-l] Re: [obm-l] Problema - Combinatória

2003-12-07 Thread David M. Cardoso








Obrigado a todos pelas respostas...

Só corrigindo o fim da tabela:

 

 
20 75186    353   576

  


  20
55111    167   223

 

 
1
41020     3556
56     56

 

  1 3
6 10 15   
21   

 


 
1 2
3 4 
 5  6 

 

 
1 1 1
1   1  1

 






=
Instruções para entrar na lista, sair da lista e usar a lista em
http://www.mat.puc-rio.br/~nicolau/olimp/obm-l.html
=

[obm-l] RE: [obm-l] 2 dúvidas

2003-12-11 Thread David M. Cardoso










 1) Seja An=6^n+8^n. Determine o resto da divisão de
A83 por 49





a)5





b)13





c)27





d)35 <<<





e)42





 

Bem.. O Domingos Jr. andou me explicando
umas coisas de congruência...

Acabou dando muito trabalho, mas deu pra
fazer assim:

 

A[83] = 6^83+8^83

 

2^83 * 3^83 + 2^249 ~ x (mod 49)

    ^^^

2^6 ~ 15 (mod 49)

2^249 ~ 2^246 * 2^3 ~ (2^6)^41 * 2^3 ~ 15^41
* 2^3 (mod 49)

15^41 * 2^3 ~ (15^2)^20 * 15 * 2^3 (mod
49)

Mas 15^2 ~ 225 ~ 29 (mod 49)

29^20 * 15 * 2^3 ~ ((29^2)^2)^5) * 3 * 5 *
2^3 ~ ((29^2)^2)^5) * 22 ~ 22^2 ~ 43 (mod 49) 

Em outras palavras:

2^249 deixa resto 43 quando dividido por
49. (Vamos guardar esse valor.)

 

2^83 ~ (2^6)^13 * 2^5 ~ 15^13 * 2^5 ~ (((15^2)^2)^3)
* 15 * 2^5 ~ ((29^2)^3) * 39 ~ 22 * 39 ~ 25 (mod 49)

2^83 ~ 25 (mod 49) (Vamos guardar esse
valor também.)

 

3^4 ~ 32 (mod 49)

3^83 ~ (3^4)^20 * 3^3 ~ ((32)^2)^10 * 27 ~
(1024)^10 * 27 ~ 44^10 * 27 ~ ... ~ 33 (mod 49)

 

2^83 * 3^83 + 2^249 ~ 25 * 33 + 43 ~ 41 +
43 ~ 84 ~ 35 (mod 49)

 

Hm... acho q o resto dah 35...










RE: [obm-l] Identidades de mdc

2004-03-06 Thread David M. Cardoso
Eu nao sei muita coisa sobre esse negocio,
Mas depois q eu li a mensagem do Fábio deu vontade
de tentar fazer esse seguindo o mesmo raciocinio.. ;)
[tomara q esteja certo]

> mdc(a,b) = mdc(a,a+b)

Seja d = (a,b)

a = di
b = dj

(a,a+b) = (di, di + dj) = d(i, i+j)

p = (i, i+j)

i = pk
i+j = pq
j = p(q-k)

como vimos, i e j sao primos entre si, logo p=1
i = k
j = q-k
i+j = q

(i, i+j) = (k, q)

Como vimos, k e q sao primos entre si, logo (i, i+j) = 1


=
Instruções para entrar na lista, sair da lista e usar a lista em
http://www.mat.puc-rio.br/~nicolau/olimp/obm-l.html
=


[obm-l] RE: [obm-l] Combinatória

2004-03-08 Thread David M. Cardoso

http://www.mat.puc-rio.br/~nicolau/olimp/obm-rj.1998/msg00013.html

> -Original Message-
> From: [EMAIL PROTECTED] [mailto:[EMAIL PROTECTED] On
> Behalf Of [EMAIL PROTECTED]
> Sent: segunda-feira, 8 de março de 2004 02:03
> To: [EMAIL PROTECTED]
> Subject: [obm-l] Combinatória
> 
> Alguem poderia me ajudar nesse problema?
> 
> Um grupo de 8 cientistas trabalha em um projeto altamente sigiloso, cujos
> planos estão guardados em um armário. Eles desejam que o armário só possa
> ser aberto quando pelo menos 5 ciêntistas estiverem presentes. Para que
> isso aconteça, são instalados cadeados no armário e cada cientista recebe
> as chaves de alguns cadeados. Suponha que tenha sido instalada a menor
> quantidade possível de cadeados:
> 
> Quantos cadeados foram instalados?
> (a) 8 (b) 28 (c)56 (d) 64 (e) 70
> 
> Qauntas chaves cada cientista recebeu?
> (A)2 (b)4 (c) 21 (d) 32 (e) 35
> 
> grato
> Junior


=
Instruções para entrar na lista, sair da lista e usar a lista em
http://www.mat.puc-rio.br/~nicolau/olimp/obm-l.html
=


[obm-l] RE: [obm-l] equação modular - ratificando

2004-03-09 Thread David M. Cardoso

x^2  - 3 = [+/-]k
x = [+/-] raiz(3 [+/-] k)

x = + raiz(3 + k)
x = - raiz(3 + k)
x = + raiz(3 - k)
x = - raiz(3 - k)

Pra ter 3 soluções, temos 2 possibilidades:
1. (3 + k) = 0 e (3-k) <> 0 ==> k = (-3)
2. (3 + k) <> 0 e (3-k) = 0 ==> k = (+3)

k = {+3,-3}

???


> -Original Message-
> From: [EMAIL PROTECTED] [mailto:[EMAIL PROTECTED] On
> Behalf Of Daniel Silva Braz
> Sent: terça-feira, 9 de março de 2004 11:21
> To: [EMAIL PROTECTED]
> Subject: [obm-l] equação modular - ratificando
> 
> a equação é |x^2 – 3| = k
> 
> 
>  --- Daniel Silva Braz <[EMAIL PROTECTED]> escreveu:
> > Determine o(s) valor(es) de k para que a equação
> > |x^2 – 3| = k  tenha 3 soluções
> >
> > resolvi a equação graficamente e verifiquei que 3
> > soluções só é possível se k = 4 (entendendo q k é um
> > número real qq e não um polinomio), mas no entanto o
> > livro me deu a resposta como sendo 3
> >
> > Alguém poderia me ajudar e me dizer se estou certo
> > ou
> > não?
> >
> > Daniel S. Braz
> >
> >
> __
> >
> > Yahoo! Mail - O melhor e-mail do Brasil! Abra sua
> > conta agora:
> > http://br.yahoo.com/info/mail.html
> >
> =
> > Instruções para entrar na lista, sair da lista e
> > usar a lista em
> > http://www.mat.puc-rio.br/~nicolau/olimp/obm-l.html
> >
> =
> 
> __
> 
> Yahoo! Mail - O melhor e-mail do Brasil! Abra sua conta agora:
> http://br.yahoo.com/info/mail.html
> =
> Instruções para entrar na lista, sair da lista e usar a lista em
> http://www.mat.puc-rio.br/~nicolau/olimp/obm-l.html
> =


=
Instruções para entrar na lista, sair da lista e usar a lista em
http://www.mat.puc-rio.br/~nicolau/olimp/obm-l.html
=


[obm-l] RE: [obm-l] Duvida de inequação

2004-03-09 Thread David M. Cardoso
(x^2 - 3x + 2)(x+3)

  (a)   <+[0]++[1]--[2]>
  (b)   <[-3]+[0]++>
(a)x(b) <[-3]+[0]++[1]--[2]>

hehe.. esses (a),(b),(a)x(b) ateh q ficaram engracado..
mas continuando:

x < (-3) ou 1 < x < 2


> -Original Message-
> From: [EMAIL PROTECTED] [mailto:[EMAIL PROTECTED] On
> Behalf Of Fabio Contreiras
> Sent: terça-feira, 9 de março de 2004 21:55
> To: [EMAIL PROTECTED]
> Subject: [obm-l] Duvida de inequação
> 
> Ache o intervalo no qual :
> 
> (x-1)(x-2)(x+3) < 0
> 
> 
> abraços!!!


=
Instruções para entrar na lista, sair da lista e usar a lista em
http://www.mat.puc-rio.br/~nicolau/olimp/obm-l.html
=


[obm-l] p < sqrt(n)

2004-03-09 Thread David M. Cardoso

oi...

É possível dizer que se p é um primo que divide n, então p < sqrt(n), certo?
Se isso está certo, é fácil demonstrar?


=
Instruções para entrar na lista, sair da lista e usar a lista em
http://www.mat.puc-rio.br/~nicolau/olimp/obm-l.html
=


[obm-l] Mathematica x Maple x Mat-Lab

2004-03-10 Thread David M. Cardoso

Pra quem conhece esses softwares:

Qual deles ganha em usabilidade?
Quero dizer: facilidade de uso, interface intuitiva, etc.

=
Instruções para entrar na lista, sair da lista e usar a lista em
http://www.mat.puc-rio.br/~nicolau/olimp/obm-l.html
=


[obm-l] RE: [obm-l] Re: [obm-l] Re: [obm-l] Re: [obm-l] Seqüência

2004-03-13 Thread David M. Cardoso

[...]
> 
> 1) O MAPLE reconhece apenas 1 polinomio para alguns termos iniciais de uma
> sequencia ? Pois a sequencia 2,3,5,7,11,13,17,19,23,29,31, pode ser
> ... Por que ele (MAPLE) diz que os proximos termos sao 838, 8440, 48141,
> 200229,
> 677006, 1972016, 5126743, 12178361, 26874761, ... ? Seriam as raizes ?
> Seria muito maçante eu substituir no polinomio para ver isso...
> 
[...]

x=1 <--> p=2
x=2 <--> p=3
x=3 <--> p=5
x=4 <--> p=7
x=5 <--> p=11
...
Isso funciona até x=11... qdo ele retorna 31


=
Instruções para entrar na lista, sair da lista e usar a lista em
http://www.mat.puc-rio.br/~nicolau/olimp/obm-l.html
=


RE: [obm-l] Duvida . analise

2004-03-14 Thread David M. Cardoso

Eu vou arriscar...
Cada número formado, pode ser escrito como:

A.10^4 + B.10^3 + C.10^2 + D.10 + E

Vamos analisar o "A.10^4"

1 x 10^4 (aparece 4! = 24 vezes)
3 x 10^4 (aparece 4! = 24 vezes)
5 x 10^4 (aparece 4! = 24 vezes)
7 x 10^4 (aparece 4! = 24 vezes)
9 x 10^4 (aparece 4! = 24 vezes)

Botando 10^4 em evidencia, temos:
24x(1+3+5+7+9)x10^4
(24x25)x10^4 = 600*(10^4) = 6x10^6

O mesmo raciocinio se aplica para B,C,D e E...
onde teriamos respectivamente:
6x10^5, 6x10^4, 6x10^3, 6x10^2

Entao o resultado dessa soma é 600.

6x10^6 < 6, x 10^6 < 7x10^6
letra B ?

> -Original Message-
> From: [EMAIL PROTECTED] [mailto:[EMAIL PROTECTED] On
> Behalf Of Fabio Contreiras
> Sent: domingo, 14 de março de 2004 11:37
> To: [EMAIL PROTECTED]
> Subject: [obm-l] Duvida . analise
> 
> Considere todos os numeros de cinco algarismos formados pela justaposicao
> de 1,3,5,7,9 em qualquer ordem, sem repeticao. A soma de todos esses
> numeros esta entre
> 
> a ) 5 x 10^6 e 6 x 10^6
> b) 6 x 10^6 e 7 x 10^6
> c ) 7 x 10^6 a 8x 10^6
> d ) idem so que de 9 a 10
> e ) idem so que de 10 a 11
> 
> alguem sabe fazer isso?


=
Instruções para entrar na lista, sair da lista e usar a lista em
http://www.mat.puc-rio.br/~nicolau/olimp/obm-l.html
=


RE: [obm-l] Duvida . analise

2004-03-14 Thread David M. Cardoso

Achei muito interessante essa forma de solucionar a questão...
mas não entendi porque isso funciona, será que alguém podia explicar isso
melhor?

> -Original Message-
> From: [EMAIL PROTECTED] [mailto:[EMAIL PROTECTED] On
> Behalf Of Angelo Barone Netto
> Sent: domingo, 14 de março de 2004 18:51
> To: [EMAIL PROTECTED]
> Subject: Re: [obm-l] Duvida . analise
> 
> Caro Fabio Contreiras <[EMAIL PROTECTED]>:
> 
> 
> A sua resposta nao so esta correta como ha quem a considere muito
> elegante.
> Nao sei o que significa maceteada, razao por que me abstenho de opinar se
> e
> o caso.
> A cada um dos 120 numeros faca corresponder "outro" (dos 120 numeros)que e
> formado pelos "suplementos" (10-x) de seus algarimos na mesma ordem.
> Verifique que e "outro".
> Identifique os peres (a,b) e (b,a).
> Assim V. obtem 60 pares (com todos os 120 numeros), cada um dos
> quais soma 10.
> Logo a soma dos 120 e
>  60 x 10 =  6, x 10^6.
> Resposta
>  b) 6 x 10^6 e 7 x 10^6
> Angelo Barone Netto <[EMAIL PROTECTED]>
> =
> Instruções para entrar na lista, sair da lista e usar a lista em
> http://www.mat.puc-rio.br/~nicolau/olimp/obm-l.html
> =


=
Instruções para entrar na lista, sair da lista e usar a lista em
http://www.mat.puc-rio.br/~nicolau/olimp/obm-l.html
=


[obm-l] Somatório da função

2004-03-16 Thread David M. Cardoso

Dada a função:
f(i,n) = -(1/2)(i-n-1)(i+n)

Preciso encontrar g(n) tal que:
g(n) = f(1,n) + f(2,n) + f(3,n) + ... f(n,n)

Quem é g(n) ?


=
Instruções para entrar na lista, sair da lista e usar a lista em
http://www.mat.puc-rio.br/~nicolau/olimp/obm-l.html
=


[obm-l] RES: [obm-l] Re: [obm-l] Somatório da função

2004-03-16 Thread David M. Cardoso
> 
> Vou usar
> SOMA_{1 <= i <= n} i = n(n+1)/2
> SOMA_{1 <= i <= n} i^2 = n(n+1)(2n+1)/3
> 
> g(n) = (1/2)* SOMA_{1 <= i <= n} (n+1-i)(n+i)
> = (1/2) * SOMA (n^2 + n - in + in + i - i^2)
> = (1/2) * (n^3 + n^2 + (n(n+1)/2) - (n(n+1)(2n+1)/3))
> 

Entendi... eu entendi! Obrigado ;)


=
Instruções para entrar na lista, sair da lista e usar a lista em
http://www.mat.puc-rio.br/~nicolau/olimp/obm-l.html
=


[obm-l] RES: [obm-l] Re: [obm-l] Re: [obm-l] Somatório da função

2004-03-16 Thread David M. Cardoso
>>  Soma[i^2] = n(n+1)(2n+1)/6

Na verdade eu só entendi pq abstraí isso... e isso eu não entendi.


=
Instruções para entrar na lista, sair da lista e usar a lista em
http://www.mat.puc-rio.br/~nicolau/olimp/obm-l.html
=


[obm-l] RES: [obm-l] Somatório da função

2004-03-16 Thread David M. Cardoso
Agora eu entendi tudo... muito obrigado! 

=
Instruções para entrar na lista, sair da lista e usar a lista em
http://www.mat.puc-rio.br/~nicolau/olimp/obm-l.html
=


[obm-l] Quadrados no tabuleiro

2004-03-16 Thread David M. Cardoso

Quantos quadrados existem num tabuleiro formado por 8x8 quadradinhos?
E num tabuleiro n x n?

=
Instruções para entrar na lista, sair da lista e usar a lista em
http://www.mat.puc-rio.br/~nicolau/olimp/obm-l.html
=


RES: [obm-l] Quadrados no tabuleiro

2004-03-16 Thread David M. Cardoso
> soma(1<=i<=n)= i^2

Qual o raciocínio q usa pra chegar nisso?
Fiz uma confusão muito grande pra chegar nessa resposta... :-(


=
Instruções para entrar na lista, sair da lista e usar a lista em
http://www.mat.puc-rio.br/~nicolau/olimp/obm-l.html
=


RES: RES: [obm-l] Quadrados no tabuleiro

2004-03-16 Thread David M. Cardoso
> ...
> 
> Deu pra entender?
> 

deu sim, perfeitamente, obrigado.

=
Instruções para entrar na lista, sair da lista e usar a lista em
http://www.mat.puc-rio.br/~nicolau/olimp/obm-l.html
=


RES: [obm-l] Duvidas

2004-03-18 Thread David M. Cardoso

Não tenho tanta certeza, mas faria assim:

x/18 = 1 - (5/15 + 5/18)
x = 7

a explicação seria que se uma torneira enche 1 tanque em 18 horas.. então em
5 horas ela enche 5/18 de tanque..

> -Mensagem original-
> De: [EMAIL PROTECTED] [mailto:[EMAIL PROTECTED] Em nome
> de aryqueirozq
> Enviada em: quinta-feira, 18 de março de 2004 23:43
> Para: [EMAIL PROTECTED]
> Assunto: [obm-l] Duvidas
> 
> 
> 
> Um tanque tem duas torneiras. A primeira enche o tanque
> em 15 horas
>   e a segunda em 18 horas. Estando o tanque vazio e
> abrindo-se as duas
> torneiras durante as primeiras 5 horas , enche-se uma
> parte do tanque.
> Podemos afirmar que , a segunda torneira encherá o
> restante do tanque em :
> a)7 horas
> b)8 horas
> c)13 horas
> d)10 horas
> e)8,5 horas
> 
> 
> 
> Agradeço desde de já.
> 
> 
> __
> Acabe com aquelas janelinhas que pulam na sua tela.
> AntiPop-up UOL - É grátis!
> http://antipopup.uol.com.br/
> 
> 
> 
> =
> Instruções para entrar na lista, sair da lista e usar a lista em
> http://www.mat.puc-rio.br/~nicolau/olimp/obm-l.html
> =


=
Instruções para entrar na lista, sair da lista e usar a lista em
http://www.mat.puc-rio.br/~nicolau/olimp/obm-l.html
=


RES: [obm-l] P.A

2004-03-21 Thread David M. Cardoso
> a5 + a8 = 130 e a4 + a10 = 140

I)  A1 + 4r + A1 + 7r = 130
II) A1 + 3r + A1 + 9r = 140

r  = 10
A1 = 10

> Qual é a P.A finita em que o primeiro termo é 8, o último termo é 38, e o
> números de termos é igual a razão?

38 = 8 + (r-1)r
30 = (r-1)r

r = 6


=
Instruções para entrar na lista, sair da lista e usar a lista em
http://www.mat.puc-rio.br/~nicolau/olimp/obm-l.html
=


[obm-l] RES: [obm-l] dúvida

2004-03-26 Thread David M. Cardoso

Colocando esse sistema no mathematica:

Tg[Pi/3] = h / x
Tg[Pi/6] = h / (x+d)

ele retorna:
d = 2*h*raiz(3)/3

> Um caçador avista um pato voando em direção horizontal, a uma altura "h"
> do solo. Inclina sua arma 60º e dá o primeiro disparo, que atinge a ave de
> raspão; abaixa a arma para 30º e dá o segundo disparo que atinge a ave em
> cheio. A distância percorrida pela ave, do primeiro ao segundo
> disparo,supondo que manteve o vôo na horizontal foi de:
> a)30 b)2h c)[2h*raiz(3)]/3 d)h/3 e)raiz(3)/3


=
Instruções para entrar na lista, sair da lista e usar a lista em
http://www.mat.puc-rio.br/~nicolau/olimp/obm-l.html
=


[obm-l] RES: RES: [obm-l] dúvida

2004-03-27 Thread David M. Cardoso

> Suponho que h seja a altura de um eixo imaginario (perpendicular ao solo)
> que vai do solo ateh a bala,

Na verdade eu entendi H como sendo a altura [...] q vai do solo ateh o pato.

> ... passando pelo corpo da ave, certo ? Logo h eh
> crescente no intervalo [0,90º[, ou seja, a inclinacao da arma produz um
> angulo alfa, cujo cateto oposto eh h, certo ? Entao quanto maior for alfa
> (de 0 a 90, eh claro) maior sera h. Mas vejamos o sistema:
> 
> Tg[Pi/3] = h / x
> Tg[Pi/6] = h / (x+d)
> 
> Porque o h da primeira equacao eh o mesmo da segunda equacao ? As alturas
> nao serao diferentes de acordo com a declividade da arma ?

Eu entendi q o pato voava no sentido horizontal, paralelo ao chão e
perpendicularmente ao eixo de H. se for assim mesmo, a altura do pato não
varia.

Por esse raciocínio, x seria a distancia, na horizontal, do atirador até o
pato no primeiro disparo. E "d" seria a distância que o pato se afastou
(sempre na horizontal) entre o primeiro e o segundo disparo.

> 
> 
> 
> 
> 
> 
> Em uma mensagem de 27/3/2004 03:18:14 Hora padrão leste da Am. Sul,
> [EMAIL PROTECTED] escreveu:
> 
> 
> 
> 
>   e escrevendo no papel:
> 
>   Tg[Pi/3] = h / x
>   Tg[Pi/6] = h / (x+d)
> 
>   sqrt(3) = h / x  ==> x = h/sqrt(3)
>   sqrt(3)/3 = h / (x+d)
> 
>   sqrt(3)/3 = h / ((h/sqrt(3)+d)
>   h + d.sqrt(3) = 3h
>   d = 2h/sqrt(3)
>   d = 2h.sqrt(3)/3
> 
>   bom, o papel retorna o mesmo resultado.. rs
> 
> 
> 
>   David M. Cardoso escreveu:
> 
>   >
>   > Colocando esse sistema no mathematica:
>   >
>   > Tg[Pi/3] = h / x
>   > Tg[Pi/6] = h / (x+d)
>   >
>   > ele retorna:
>   > d = 2*h*raiz(3)/3
>   >
>   >> Um caçador avista um pato voando em direção horizontal, a uma
> altura "h"
>   >> do solo. Inclina sua arma 60º e dá o primeiro disparo, que atinge
> a ave de
>   >> raspão; abaixa a arma para 30º e dá o segundo disparo que atinge
> a ave em
>   >> cheio. A distância percorrida pela ave, do primeiro ao segundo
>   >> disparo,supondo que manteve o vôo na horizontal foi de:
>   >> a)30 b)2h c)[2h*raiz(3)]/3 d)h/3 e)raiz(3)/3
>   >
>   >
> 
> 
> 
> 



=
Instruções para entrar na lista, sair da lista e usar a lista em
http://www.mat.puc-rio.br/~nicolau/olimp/obm-l.html
=


[obm-l] RES: RES: [obm-l] dúvida

2004-03-27 Thread David M. Cardoso

Heheh.. vixe maria... êta galerinha pra gostah de complicar: qdo o problema
eh fácil sempre tem um pra comecar a botar detalhe ateh não saber mais -
hehe...

Pootz:
h' = h + [altura da arma em relação ao chão]

onde chão é o "eixo imaginário bla bla bla" . rs

> -Mensagem original-
> De: [EMAIL PROTECTED] [mailto:[EMAIL PROTECTED] Em nome
> de [EMAIL PROTECTED]
> Enviada em: sábado, 27 de março de 2004 16:51
> Para: [EMAIL PROTECTED]
> Assunto: Re: RES: [obm-l] dúvida
> 
> Me chama atenção que não está sendo considerado o fato do homem não estar
> com a arma nos pés...
> 
> ariel ([EMAIL PROTECTED]) escreveu:
> >
> >e escrevendo no papel:
> >
> >Tg[Pi/3] = h / x
> >Tg[Pi/6] = h / (x+d)
> >
> >sqrt(3) = h / x  ==> x = h/sqrt(3)
> >sqrt(3)/3 = h / (x+d)
> >
> >sqrt(3)/3 = h / ((h/sqrt(3)+d)
> >h + d.sqrt(3) = 3h
> >d = 2h/sqrt(3)
> >d = 2h.sqrt(3)/3
> >
> >bom, o papel retorna o mesmo resultado.. rs
> >
> >
> >
> >David M. Cardoso escreveu:
> >
> >>
> >> Colocando esse sistema no mathematica:
> >>
> >> Tg[Pi/3] = h / x
> >> Tg[Pi/6] = h / (x+d)
> >>
> >> ele retorna:
> >> d = 2*h*raiz(3)/3
> >>
> >>> Um caçador avista um pato voando em direção horizontal, a uma altura
> "h"
> >>> do solo. Inclina sua arma 60º e dá o primeiro disparo, que atinge a
> ave
> de
> >>> raspão; abaixa a arma para 30º e dá o segundo disparo que atinge a ave
> em
> >>> cheio. A distância percorrida pela ave, do primeiro ao segundo
> >>> disparo,supondo que manteve o vôo na horizontal foi de:
> >>> a)30 b)2h c)[2h*raiz(3)]/3 d)h/3 e)raiz(3)/3
> >>
> >>
> >>
> =
> >> Instruções para entrar na lista, sair da lista e usar a lista em
> >> http://www.mat.puc-rio.br/~nicolau/olimp/obm-l.html
> >>
> =
> >
> >=
> >Instruções para entrar na lista, sair da lista e usar a lista em
> >http://www.mat.puc-rio.br/~nicolau/olimp/obm-l.html
> >=
> >
> 
> =
> Instruções para entrar na lista, sair da lista e usar a lista em
> http://www.mat.puc-rio.br/~nicolau/olimp/obm-l.html
> =


=
Instruções para entrar na lista, sair da lista e usar a lista em
http://www.mat.puc-rio.br/~nicolau/olimp/obm-l.html
=


RES: [obm-l] inequacao do 2 grau

2004-03-28 Thread David M. Cardoso
Pareceu estranho qdo eu acabei, mas serah q eh isso aki: ??

x^2 - (3+k)x = 0

x(x - (3+k)) = 0
x'  = 0
x'' = 3+k

k != -3

?

> -Mensagem original-
> De: [EMAIL PROTECTED] [mailto:[EMAIL PROTECTED] Em nome
> de Guilherme Teles
> Enviada em: domingo, 28 de março de 2004 21:24
> Para: [EMAIL PROTECTED]
> Assunto: [obm-l] inequacao do 2 grau
> 
> Alguem sabe essa:
> determine k para que a funcao real y=x^2-(3+k)x admita dois zeros reais e
> diferentes.


=
Instruções para entrar na lista, sair da lista e usar a lista em
http://www.mat.puc-rio.br/~nicolau/olimp/obm-l.html
=


[obm-l] CR(n,p) = C(n+p-1,p)

2004-03-29 Thread David M. Cardoso

CR -> Combinação com repetição.
C  -> Combinação

Se não me engano, existe uma formula q diz o seguinte:
CR(n,p) = C(n+p-1,p)

Eu queria entender a lógica dessa formula,
que é útil por exemplo pra resolver:

a + b + c + d = 20
Quantas soluções tem isso?

Exemplos de soluções:
3 + 4 + 2 + 11 => 111..11.111
5 + 0 + 9 + 6  => 1..1.11

Para calcular todas as soluções, seria (eu acho) "só"
contar de quantas formas eu posso posicionar 3 pontinhos
em 21 posições.

O meu conceito de Combinação era decorado, até eu ler
uma mensagem do Nicolau, explicando como se chegava
na fórmula..
(http://www.mail-archive.com/[EMAIL PROTECTED]/msg18766.html)
Agora queria entender a CR.


=
Instruções para entrar na lista, sair da lista e usar a lista em
http://www.mat.puc-rio.br/~nicolau/olimp/obm-l.html
=


RES: [obm-l] CR(n,p) = C(n+p-1,p)

2004-03-29 Thread David M. Cardoso
> [...]
> Mas isso eh igual a Binom(n+p-1,p-1).
> 
> Espero ter sido claro.
> 
> 
> []s,
> Claudio.

Cara... às vezes eu fico ateh emocionado com as mensagens
de voces... que explicação do kct!!! entendido..
Valeu mesmo!

David


=
Instruções para entrar na lista, sair da lista e usar a lista em
http://www.mat.puc-rio.br/~nicolau/olimp/obm-l.html
=


RES: [obm-l] CR(n,p) = C(n+p-1,p)

2004-03-29 Thread David M. Cardoso
> SUGESTÃO:  Leia o livro do Morgado:
> Análise Combinatória e Probabilidade, Coleção do Professor de Matemática
> Capítulo 2, seção 2.6 (Combinações Completas).
> Além de uma leitura fácil, tem uma coleção de problemas muito
> interessante,
> ao mesmo tempo desafiadora e motivadora.
> No final do livro, você encontra uma bibliografia comentada, sobre Análise
> Combinatória,
> que é uma das melhores coisas que já lí.
> Experimente! Vale a pena!
> Benedito

Onde eu compro? Você recomenda algum lugar pra eu comprar?


=
Instruções para entrar na lista, sair da lista e usar a lista em
http://www.mat.puc-rio.br/~nicolau/olimp/obm-l.html
=


RES: RES: [obm-l] CR(n,p) = C(n+p-1,p)

2004-03-29 Thread David M. Cardoso

Bem.. eu moro em Recife/PE.. se não tiver como eu comprar por aqui,
eu vo ter q esperar meu pai ir pra sp, assim ele traz pra mim, mas se esse
for o caso eu vou precisar saber dizer pra ele onde ele tem que ir pra
comprar esse livro pra mim..

Agradeço muito a boa vontade..
David


=
Instruções para entrar na lista, sair da lista e usar a lista em
http://www.mat.puc-rio.br/~nicolau/olimp/obm-l.html
=


RES: [obm-l] Problema de Torneiras

2004-03-30 Thread David M. Cardoso

(1/4) + (x/25) + (x/40) - (x/20) = 1

[... contas ...]

x = 50 horas


Princípio:

"A primeira enxe o tanque em 25 horas"

25 -- 1


> -Mensagem original-
> De: [EMAIL PROTECTED] [mailto:[EMAIL PROTECTED] Em nome
> de Fabio Contreiras
> Enviada em: terça-feira, 30 de março de 2004 23:08
> Para: [EMAIL PROTECTED]
> Assunto: [obm-l] Problema de Torneiras
> 
> Esse é maneiro! Alguem sabe o caminho das pedras?
> 
> 
>  1 ) Um tanque tem 3 torneiras. A primeira enxe o tanque em 25 horas, a
> segunda em 40 horas, ja a terceira, o esvazia em 20 horas. O tanque está
> com 1 / 4 de água. Abrindo-se simultaneamente as três torneiras, ele
> ficará cheio em :
> 
> 


=
Instruções para entrar na lista, sair da lista e usar a lista em
http://www.mat.puc-rio.br/~nicolau/olimp/obm-l.html
=


RES: [obm-l] Problema de Torneiras

2004-03-30 Thread David M. Cardoso

Apertei control+enter e enviei a mensagem sem querer (desculpa!),
continuando:

> (1/4) + (x/25) + (x/40) - (x/20) = 1
> 
> [... contas ...]
> 
> x = 50 horas
> 
> 
> Princípio:
> 
> "A primeira enCHe o tanque em 25 horas"

25 horas -> 1 tanque
x horas  -> (x/25) tanque.


=
Instruções para entrar na lista, sair da lista e usar a lista em
http://www.mat.puc-rio.br/~nicolau/olimp/obm-l.html
=


RES: [obm-l] PA

2004-04-05 Thread David M. Cardoso


(a - r) + a + (a + r) = 18
3a = 18
a = 6

1/(6-r) + 1/6 + 1/(6+r) = 23/30

[... contas ...]

r = 4

S = { 2, 6, 10 }


> -Mensagem original-
> De: [EMAIL PROTECTED] [mailto:[EMAIL PROTECTED] Em nome
> de Guilherme Teles
> Enviada em: segunda-feira, 5 de abril de 2004 23:00
> Para: [EMAIL PROTECTED]
> Assunto: [obm-l] PA
> 
> alguem sabe resolver essa,
> obtenha 3 numeros em PA, sabendo que sua soma é 18 e a soma de seus
> inversos é 23/30
> 


=
Instruções para entrar na lista, sair da lista e usar a lista em
http://www.mat.puc-rio.br/~nicolau/olimp/obm-l.html
=


[obm-l] RES: [obm-l] RES: [obm-l] Forma ção dos participantes da lista

2004-04-16 Thread David M. Cardoso

Ciência da Computação (1o. período) na UFPE
Adoro matemática, adoro essa lista, adoro vocês.. (hehe) ":-)
Um dia eu vou saber um monte de coisa tb, aih vou contribuir mais..
(tudo bem... eu sei.. offtopic bla bla bla... eh soh um desabafo.)

falow


=
Instruções para entrar na lista, sair da lista e usar a lista em
http://www.mat.puc-rio.br/~nicolau/olimp/obm-l.html
=


[obm-l] RES: [obm-l] provas do IME(ROGÉRIO) ?

2004-04-19 Thread David M. Cardoso

¬¬
tsc tsc 

> -Mensagem original-
> De: [EMAIL PROTECTED] [mailto:[EMAIL PROTECTED] Em nome
> de TSD
> Enviada em: domingo, 18 de abril de 2004 14:20
> Para: [EMAIL PROTECTED]
> Assunto: [obm-l] provas do IME(ROGÉRIO) ?
> 
> A DIGITAÇÃO TÁ MUITO CEBOZA. NUM TERIA COMO MELHORAR A QUALIDADE DA
> ESCRITA NOS ENUNCIADOS E RESOLUÇÕES ?
> 


=
Instruções para entrar na lista, sair da lista e usar a lista em
http://www.mat.puc-rio.br/~nicolau/olimp/obm-l.html
=


RES: [obm-l] Algoritmo

2004-05-14 Thread David M. Cardoso

Um algoritmo muito ineficiente (mas que pra n < 50 isso não faz a **menor**
a diferenca)
mas ao mesmo tempo muito facil de entender eh esse: (chamado de
bubblesort)..

double medias[0..9];
int i = 1
int n = 10
double aux = 0.0;

while (i < n)
{
   if (medias[i] > medias[i-1])
   {
 aux = medias[i-1];
 medias[i-1] = medias[i];
 medias[i] = aux;
 
 i=1;
   }
   else
 i++;
}

Um algoritmo que eu acho bem eficiente, mas eh um pouco mais dificil de
entender eh o
"quicksort"... se vc tiver interesse, uma busca no google vai achar zilhoes
de paginas
com exemplos.

Falow...
David.

> -Mensagem original-
> De: [EMAIL PROTECTED] 
> [mailto:[EMAIL PROTECTED] Em nome de rickufrj
> Enviada em: quinta-feira, 13 de maio de 2004 23:20
> Para: obm-l
> Assunto: [obm-l] Algoritmo
> 
> Olá pessoal , será que alguém poderia me ajudar nesse algoritmo :
> 
> Fazer um algoritmo que leia a media de 10 alunos e em seguida 
> as coloque em ordem decrescente.
> 
> Eu to me enrolando um pouco , pois não consigo montar um 
> algoritmo compacto ,nas idéias que eu tive , estava 
> descambando para um lado muito grande  .
> Será que alguém poderia me ajudar utilizando os loop’s for ou 
> outro tipo de loop ?
> Agradeço a atenção
> Abraços
> Luiz H. Barbosa
>  
> __
> 
> Acabe com aquelas janelinhas que pulam na sua tela.
> AntiPop-up UOL - É grátis!
> http://antipopup.uol.com.br/
> 
> 
> 
> ==
> ===
> Instruções para entrar na lista, sair da lista e usar a lista 
> em http://www.mat.puc-rio.br/~nicolau/olimp/obm-l.html
> ==
> ===
> 


=
Instruções para entrar na lista, sair da lista e usar a lista em
http://www.mat.puc-rio.br/~nicolau/olimp/obm-l.html
=


[obm-l] RES: [obm-l] Dúvidas: Algoritimo

2004-05-14 Thread David M. Cardoso

Bem.. não eh aquela-eficiencia, mas foi o melhor q eu pude pensar..

bool temElementosRepetidos(int[] arr, int n)
{
   bool temElementosRepetidos = false;
   int elementoProcurado = 0;   

   for (int i=0; i -Mensagem original-
> De: [EMAIL PROTECTED] 
> [mailto:[EMAIL PROTECTED] Em nome de Marcos Eike
> Enviada em: quinta-feira, 14 de outubro de 2004 00:24
> Para: [EMAIL PROTECTED]
> Assunto: [obm-l] Dúvidas: Algoritimo
> 
> Pessoal,
>  
> eu estou querendo criar um algoritmo para analisar cada 
> entrada de um numero, num array de 5 elementos, comparando 
> com os elementos anteriores. Sendo que com isso eu consiga 
> assegurar que esse array nao tera elementos repetidos..
>  
> Basta me mostrar um possivel caminho... dum jeito mais 
> optimizado possivel 
>  
>  
> Obrigado!
>  
> 


=
Instruções para entrar na lista, sair da lista e usar a lista em
http://www.mat.puc-rio.br/~nicolau/olimp/obm-l.html
=


[obm-l] Conjuntos Contáveis

2004-05-26 Thread David M. Cardoso


Como demonstrar que se A e B são contáveis, então o produto
cartesiano AxB também eh contável?


=
Instruções para entrar na lista, sair da lista e usar a lista em
http://www.mat.puc-rio.br/~nicolau/olimp/obm-l.html
=


RES: [obm-l] Soma...

2004-05-26 Thread David M. Cardoso

Extraindo dessa mensagem essa parte:

>   Seja S[n] o polinômio que representa a soma dos 
> quadrados dos n primeiros inteiros positivos, então podemos 
> concluir que:
> S[n] = S[n - 1] + n^2 => S[n] - S[n - 1] = n^2 (i)
> 
> Logo S[n] tem que ser um polinômio de grau 3, uma vez que na 
> diferença S[n]
> - S[n - 1] os termos de maior grau dos polinômios vão ser 
> cancelados.

Não entendi pq o dá pra inferir que o grau do polinomio é 3...
Será alguem pode explicar isso?
 

> -Mensagem original-
> De: [EMAIL PROTECTED] 
> [mailto:[EMAIL PROTECTED] Em nome de Rogério Moraes 
> de Carvalho
> Enviada em: quarta-feira, 19 de maio de 2004 10:25
> Para: [EMAIL PROTECTED]
> Assunto: RE: [obm-l] Soma...
> 
> Olá Crom,
> 
>   Muitos livros de Matemática apresentam uma possível 
> dedução da fórmula da soma das potências k-ésimas (k inteiro 
> positivo) dos n primeiros inteiros positivos pelo método que 
> você apresentou parcialmente, ou seja, usando o 
> desenvolvimento do binômio de Newton (x + 1)^(k + 1). Ao 
> aplicar o somatório com x variando de 1 até n a ambos os 
> membros da igualdade, os termos de grau (k + 1) podem ser 
> cancelados, com exceção de (n + 1)^(k + 1) no primeiro membro 
> da igualdade e 1^(k + 1) = 1 no segundo membro da igualdade. 
> Porém, para descobrir a fórmula da soma das potências 
> k-ésimas, nós precisamos conhecer todas as fórmulas das somas 
> das potências com expoente de 1 até (k - 1). Sendo assim, nós 
> encontramos uma fórmula de recorrência para deduzir a soma 
> das potências k-ésimas dos n primeiros inteiros positivos, 
> porém o processo vai ficando muito longo à medida que os 
> expoentes vão crescendo.
> 
>   A seguir, eu apresento um método que pode ser utilizado 
> para encontrar a soma das potências k-ésimas dos n primeiros 
> inteiros positivos de forma direta. Neste método, não há a 
> necessidade de se conhecer as fórmulas das somas das 
> potências com expoente de 1 até (k - 1)
> 
> 
> DEDUÇÃO POSSÍVEL:
> 
>   Seja S[n] o polinômio que representa a soma dos 
> quadrados dos n primeiros inteiros positivos, então podemos 
> concluir que:
> S[n] = S[n - 1] + n^2 => S[n] - S[n - 1] = n^2 (i)
> 
> Logo S[n] tem que ser um polinômio de grau 3, uma vez que na 
> diferença S[n]
> - S[n - 1] os termos de maior grau dos polinômios vão ser 
> cancelados. Sendo assim, podemos escrever:
> S[n] = a.n^3 + b.n^2 + c.n + d
> O termo independente é 0, uma vez que S[0] não possui termos. 
> Portanto, d = 0.
> S[n] = a.n^3 + b.n^2 + c.n (ii)
> 
> Substituindo a (ii) na (i):
> a.n^3 + b.n^2 + c.n - a.(n - 1)^3 - b.(n - 1)^2 - c.(n - 1) = n^2
> 3a.n^2 - 3a.n + a + 2b.n - b + c = n^2
> 3a.n^2 + (2b - 3a).n + (a - b + c) = n^2
> 
> Pela identidade de polinômios, devemos ter:
> 3a = 1 <=> a = 1/3
> 2b - 3a = 0 <=> 2b - 1 = 0 <=> b = 1/2
> a - b + c = 0 <=> 1/3 - 1/2 + c = 0 <=> c = 1/6
> 
> Substituindo a, b e c no polinômio (ii):
> S[n] = n^3/3 + n^2/2 + n/6
> 
> Fatorando:
> S[n] = (2.n^3 + 3.n^2 + n)/6
> S[n] = [n(2n^2 + 3n + 1)]/6
> 
> S[n] = [n(n + 1)(2n + 1)]/6
> 
> Para o caso particular do problema apresentado, teremos:
> S[10] = (10.11.21)/6 => S[10] = 385
> 
> 
> Atenciosamente,
> 
> Rogério Moraes de Carvalho
> 
> From: [EMAIL PROTECTED] 
> [mailto:[EMAIL PROTECTED] On Behalf Of [EMAIL PROTECTED]
> Sent: quarta-feira, 19 de maio de 2004 01:21
> To: [EMAIL PROTECTED]
> Subject: [obm-l] Soma...
> 
> Qual o valor de S=1^2+2^2+3^2+.+10^2?
> Usei para resolver esse problema a identidade (x+1)^3. Com efeito,
> 2^3=(1+1)^3=1^3+3*1^2*1+3*1*1^2+1^1
> 3^3=(2+1)^3=2^3+3*2^2*1+3*2*1^2+1
> --
> 11^3=(10+1)^3=10^3+3*10^2*1+3*10*1^2+1.Isolando 
> convenientemente 3*1^2+3*2^2++3*10^2. descubro S. Minha 
> pergunta é: Existe um modo mais fácil de se achar soma de 
> quadrados perfeitos??
>   Quem souber e puder responder, deixo meu agradecimento.
>    Crom
> 
> 
> 
> ==
> ===
> Instruções para entrar na lista, sair da lista e usar a lista 
> em http://www.mat.puc-rio.br/~nicolau/olimp/obm-l.html
> ==
> ===
> 


=
Instruções para entrar na lista, sair da lista e usar a lista em
http://www.mat.puc-rio.br/~nicolau/olimp/obm-l.html
=


RES: [obm-l] Soma...

2004-05-28 Thread David M. Cardoso

Oi, eu entendi... muito muito muito obrigado...
Achei esse negocio muito util.. não conhecia..
Obrigado de novo..

[]'s
David

> -Mensagem original-
> De: [EMAIL PROTECTED] 
> [mailto:[EMAIL PROTECTED] Em nome de Rogério Moraes 
> de Carvalho
> Enviada em: sexta-feira, 28 de maio de 2004 16:22
> Para: [EMAIL PROTECTED]
> Assunto: RE: [obm-l] Soma...
> 
> Olá David,
> 
>   Se você considerar S[n] como um polinômio de grau k em 
> n (k inteiro positivo), então:
> 
> S[n]=a[k].n^k+a[k-1].n^(k-1)+...+a[1].n+a[0], tais que a[0], 
> a[1], ..., a[k] são os coeficientes de S[n] e a[k]!=0.
> 
> S[n-1]=a[k].(n-1)^k+a[k-1].(n-1)^(k-1)+...+a[1].(n-1)+a[0]
> 
> Considerando a notação C(u, v)=u!/[v!(u-v)!], com u e v 
> inteiros não negativos e u >= v, e aplicando o 
> desenvolvimento do binômio de Newton nas expressões (n-1)^p, 
> com p pertencente a {1, 2, ..., k}, teremos:
> 
> S[n]-S[n-1] = {a[k]-C(k,0).a[k]}.n^k +
> {a[k-1]+C(k,1).a[k]-C(k-1,0).a[k-1]}.n^(k-1) + ...
> S[n]-S[n-1] = {a[k]-a[k]}.n^k + {a[k-1]+k.a[k]-a[k-1]}.n^(k-1) + ...
> S[n]-S[n-1] = k.a[k].n^(k-1) + ...
> 
> Como, por hipótese, k é inteiro positivo e a[k]!=0, então 
> k.a[k]!=0. Sendo
> assim: grau{S[n]-S[n-1]} = k-1 (i)
> Como: S[n]-S[n-1]=n^2 => grau{S[n]-S[n-1]} = 2 (ii)
> 
> Por (i) e (ii): k-1 = 2 <=> k = 3
> 
> 
> Atenciosamente,
> 
> Rogério Moraes de Carvalho
> -Original Message-
> From: [EMAIL PROTECTED] 
> [mailto:[EMAIL PROTECTED] On Behalf Of David M. Cardoso
> Sent: quarta-feira, 26 de maio de 2004 20:49
> To: [EMAIL PROTECTED]
> Subject: RES: [obm-l] Soma...
> 
> 
> Extraindo dessa mensagem essa parte:
> 
> > Seja S[n] o polinômio que representa a soma dos quadrados dos n 
> > primeiros inteiros positivos, então podemos concluir que:
> > S[n] = S[n - 1] + n^2 => S[n] - S[n - 1] = n^2 (i)
> > 
> > Logo S[n] tem que ser um polinômio de grau 3, uma vez que 
> na diferença 
> > S[n]
> > - S[n - 1] os termos de maior grau dos polinômios vão ser 
> cancelados.
> 
> Não entendi pq o dá pra inferir que o grau do polinomio é 3...
> Será alguem pode explicar isso?
>  
> 
> > -Mensagem original-
> > De: [EMAIL PROTECTED]
> > [mailto:[EMAIL PROTECTED] Em nome de Rogério Moraes de 
> > Carvalho Enviada em: quarta-feira, 19 de maio de 2004 10:25
> > Para: [EMAIL PROTECTED]
> > Assunto: RE: [obm-l] Soma...
> > 
> > Olá Crom,
> > 
> > Muitos livros de Matemática apresentam uma possível dedução da 
> > fórmula da soma das potências k-ésimas (k inteiro
> > positivo) dos n primeiros inteiros positivos pelo método que você 
> > apresentou parcialmente, ou seja, usando o desenvolvimento 
> do binômio 
> > de Newton (x + 1)^(k + 1). Ao aplicar o somatório com x 
> variando de 1 
> > até n a ambos os membros da igualdade, os termos de grau (k 
> + 1) podem 
> > ser cancelados, com exceção de (n + 1)^(k + 1) no primeiro 
> membro da 
> > igualdade e 1^(k + 1) = 1 no segundo membro da igualdade.
> > Porém, para descobrir a fórmula da soma das potências k-ésimas, nós 
> > precisamos conhecer todas as fórmulas das somas das potências com 
> > expoente de 1 até (k - 1). Sendo assim, nós encontramos uma 
> fórmula de 
> > recorrência para deduzir a soma das potências k-ésimas dos 
> n primeiros 
> > inteiros positivos, porém o processo vai ficando muito 
> longo à medida 
> > que os expoentes vão crescendo.
> > 
> > A seguir, eu apresento um método que pode ser utilizado para 
> > encontrar a soma das potências k-ésimas dos n primeiros inteiros 
> > positivos de forma direta. Neste método, não há a necessidade de se 
> > conhecer as fórmulas das somas das potências com expoente 
> de 1 até (k 
> > - 1)
> > 
> > 
> > DEDUÇÃO POSSÍVEL:
> > 
> > Seja S[n] o polinômio que representa a soma dos quadrados dos n 
> > primeiros inteiros positivos, então podemos concluir que:
> > S[n] = S[n - 1] + n^2 => S[n] - S[n - 1] = n^2 (i)
> > 
> > Logo S[n] tem que ser um polinômio de grau 3, uma vez que 
> na diferença 
> > S[n]
> > - S[n - 1] os termos de maior grau dos polinômios vão ser 
> cancelados. 
> > Sendo assim, podemos escrever:
> > S[n] = a.n^3 + b.n^2 + c.n + d
> > O termo independente é 0, uma vez que S[0] não possui termos. 
> > Portanto, d = 0.
> > S[n] = a.n^3 + b.n^2 + c.n (ii)
> > 
> > Substituindo a (ii) na (i):
> > a.n^3 + b.n^2 + c.n - a.(n - 1)^3 - b.(n - 1)^2 - c.(n - 1) = n^2
> > 3a.n^2 - 3a.n + a + 2b.n - b + c = n^2
> > 3a.n^2 + (2b - 3a).n + (a -